% % First DB, uses ran_toks and eqexam % \bRTVToks{DB1} \begin{rtVW} \begin{problem}[5] DB1: First: Which is correct? \begin{answers}{4} \bChoices[nCols=1,random,label=db1-1] \Ans1 True\eAns \Ans0 False\eAns \Ans0 Maybe\eAns \Ans0 Perhaps\eAns \eChoices \end{answers} \begin{solution} It is obvious that the answer is `\useSavedAns{db1-1}', that is, alternative \useSavedAlts{db1-1} is correct. \end{solution} \end{problem} \end{rtVW} \begin{rtVW} \begin{problem}[5] DB1: Second: Which is correct? \begin{answers}{4} \bChoices[nCols=1,random,label=db1-2] \Ans1 True\eAns \Ans0 False\eAns \Ans0 Maybe\eAns \Ans0 Perhaps\eAns \eChoices \end{answers} \begin{solution} According to the guy sitting next to me, the answer might be \useSavedAlts{db1-2} \end{solution} \end{problem} \end{rtVW} \begin{rtVW} \begin{problem}[5] DB1: Third: Which is correct? \begin{answers}{4} \bChoices[nCols=1,random,label=db1-3] \Ans1 True\eAns \Ans0 False\eAns \Ans0 Maybe\eAns \Ans0 Perhaps\eAns \eChoices \end{answers} \begin{solution} I know the answer in advance to be \useSavedAlts{db1-3}, but don't tell anyone. \end{solution} \end{problem} \end{rtVW} \begin{rtVW} \begin{problem}[5] DB1: Fourth: Which is correct? \begin{answers}{4} \bChoices[nCols=1,random,label=db1-4] \Ans1 True\eAns \Ans0 False\eAns \Ans0 Maybe\eAns \Ans0 Perhaps\eAns \eChoices \end{answers} \begin{solution} After meditating in the lotus position on this question for many hours, we determine that the correct answer is \textbf{\useSavedAltsAns{db1-4}}. \end{solution} \end{problem} \end{rtVW} \eRTVToks \endinput